Is a < b?

This topic has expert replies
Legendary Member
Posts: 2276
Joined: Sat Oct 14, 2017 6:10 am
Followed by:3 members

Is a < b?

by VJesus12 » Thu Feb 01, 2018 1:22 am
Is a < b?

(1) a^b<b^a.
(2) a/b>1\ .

The OA is the option E.

I thought statement (2) was sufficient. Why is not? Experts, may you clarify this for me? Thanks in advanced.

User avatar
Legendary Member
Posts: 2663
Joined: Wed Jan 14, 2015 8:25 am
Location: Boston, MA
Thanked: 1153 times
Followed by:128 members
GMAT Score:770

by DavidG@VeritasPrep » Thu Feb 01, 2018 10:00 am
VJesus12 wrote:Is a < b?

1) a^b < b^a.

2) a/b>1 .

The OA is the option E.

I thought statement (2) was sufficient. Why is not? Experts, may you clarify this for me? Thanks in advanced.
Statement 1: pick easy numbers. Case 1: a = -2 and b = -1. This gives us a YES, a<b
Case 2: a = 4 b = 3, This gives us a NO, a is not less than b.
1 alone is not sufficient.

Statement 2: We can use the same numbers, as -2/-1 > 1 and 4/3> 1. So we already know that we can get a YES and a NO both for statement 2 alone and when testing the statements together. The answer is E
Veritas Prep | GMAT Instructor

Veritas Prep Reviews
Save $100 off any live Veritas Prep GMAT Course